core conditions - 1 Flashcards

1
Q

PSYCHOSIS

  • what is it?
  • three clusters of symptoms?
A

an umbrella term

—> Described as experience of being out of touch with reality, struggling to distinguish what is real from what is not.

There are THREE clusters of psychotic symptoms (may have one or a combination of these):

  • hallucinations
  • delusions
  • thought disorder.

Psychosis is an umbrella term for a group of symptoms. It is not a diagnosis in itself but a feature of many other diagnoses.

—> Presence of psychosis is not confined to mental illness.

How well did you know this?
1
Not at all
2
3
4
5
Perfectly
2
Q

Definition of delusion?

A

False, unshakeable belief, despite evidence to contrary, not held by others in the same culture and held with intense personal conviction and certainty.

How well did you know this?
1
Not at all
2
3
4
5
Perfectly
3
Q

List types of delusions? 8

nb 3 are arround thoughts, the rest are more general

nb don’t need definitions yet

A
  • nihilistic
  • grandiose
  • control
  • delusional perception
  • reference delusions
  • thought insertions
  • thought withdrawal
  • thought broadcast

nb the bottom four are delusions of thought interference

nb thought echo is an auditory halluciantion and thought blocking is a thought disorder

How well did you know this?
1
Not at all
2
3
4
5
Perfectly
4
Q

Definition of:

  • nihilistic delusions?
  • grandiose delusions?
A

Nihilistic delusions

Extremely negative delusions of being dead or part of the body decaying.

Grandiose delusions

Delusion of having of a higher status or significance, special powers or a secret mission.

How well did you know this?
1
Not at all
2
3
4
5
Perfectly
5
Q

Definition of delusion of control / passivity?

A

The subject believes their thoughts, feelings and/or actions are not their own but are being imposed/controlled by an outside force

Remember in passivity experiences the subject is ‘passive’ as they believe they are being controlled by another agent

nb this is not the same as auditory hallucinations where someone is telling them to do something/threatening them, in delusions of control the person believes they are being controlled and have no agency over their own thoughts/feelings/actions

How well did you know this?
1
Not at all
2
3
4
5
Perfectly
6
Q

Definition of:

  • delusional perceptions?
  • ideas/delusions of reference?

and what’s the difference?

A

Delusional perception

The subject receives a normal perception but it’s interpreted with delusional meaning and has immense significance (e.g. I know I’m the king as I saw the traffic light turn green)

ideas of reference

The belief that innocuous events have direct personal significance to the subject (e.g. believing something on the TV is a direct message to them). With ideas of reference they take special meanings from inanimate stimuli.

so teachnically a idea of reference is a type of delusional perception

How well did you know this?
1
Not at all
2
3
4
5
Perfectly
7
Q

Describe the 3 main delusions of thought interference?

A

Thought insertions

Thoughts which are not believed to be the subject’s own infiltrate their mind. They often have bizarre explanations of how it’s happened.

Thought withdrawal

The subject believes their thoughts have been removed from their mind by an external agency.

Thought broadcast

The belief that others can hear or are aware of an individual’s thoughts (e.g. a person may believe that other people can hear or read her thoughts)

How well did you know this?
1
Not at all
2
3
4
5
Perfectly
8
Q

Definition of hallucination?

A

A perceptual experience in the absence of an object or stimulus, that appears subjectively real but uncontrollable by the patient. To the person this has the impact of a real perception and is indistinguishable. It occurs externally (not in the mind).

Can be in any sensory modality:

  • Auditory
  • visual
  • olfactory
  • gustatory
  • sensory / tactile (touch)
  • —kinaesthetic (movement)
  • —somatic (sensation within the body)

nb visual halluciantions normally always have an organic cause

How well did you know this?
1
Not at all
2
3
4
5
Perfectly
9
Q

Definition of:

  • pseudo-hallucination?
  • illusion?

how are these different from a hallucination?

A

Pseudo-hallucination

A sensory experience vivid enough to be a hallucination but recognised by the subject not to be real.

Illusion:

False perception of a real stimulus. Three types: affect, completion and pareidolia.

An illusion is based on a real sensory stimulus that is interpreted incorrectly whereas a hallucination is created by the mind without any stimulus.

hallucinations occur without a stimulus and are peceived by the patient to be real

nb don’t put effort into learning these: types of illusion:
PAREIDOLIA
Seeing shapes in inanimate objects (e.g. faces in clouds)
COMPLETION
The mind completes partial images (e.g. seeing white triangle in > image)
AFFECT
Based on current affect (e.g. see a monster in the cupboard when scared)

How well did you know this?
1
Not at all
2
3
4
5
Perfectly
10
Q

Three main types of auditory hallucination?

A

2nd person auditory hallucination:

The subject hears voices which talk to them directly (e.g. “you’re going to die, you need to kill the baby”)

3rd person auditory hallucination:

The subject hears voices talking about them in the third person or commentating on their actions (e.g. ”he’s having a drink, now he’s going to the bathroom”)

Thought echo:

The subject experiences his own thoughts as if they were being spoken aloud. The repetition may be subtly or grossly changed.

How well did you know this?
1
Not at all
2
3
4
5
Perfectly
11
Q

definitions of:

  • depersonalisation?
  • derealisation?

when can they often occur acutely?

what diagnosed with if occur chronically?

A

Depersonalisation

A change in awareness of self, in which the individual feels they’re not real and are detached from the world. They are aware this is abnormal.

Derealisation

To the subject the external world appears unreal or artificial. They’re aware this is abnormal.

Though degrees of depersonalization and derealization can happen to anyone who is subject to temporary anxiety or stress (eg during a panic attack), chronic depersonalization is more related to individuals who have experienced a severe trauma or prolonged stress/anxiety (called a dissociative disorder).

nb depersonalisation and derealisation can occur in other psychiatric conditions but if this is the predominant symptom then diagnosed with a dissociative disorder

How well did you know this?
1
Not at all
2
3
4
5
Perfectly
12
Q

Definition of thought disorder?

A

Thought Disorder:

  • An abnormality in the mechanism of thinking
  • To the observer their speech doesn’t make sense
How well did you know this?
1
Not at all
2
3
4
5
Perfectly
13
Q

What thought disorder is being described:

Loss of structured thinking. The subject seems muddled and doesn’t become clearer with further questioning, things often seem more confusing the more you ask them.

A

Loosening of associations

How well did you know this?
1
Not at all
2
3
4
5
Perfectly
14
Q

What thought disorder is being described AND what condition often seen in?

Rapid flow of thought, manifested by accelerated speech with abrupt changes from topic to topic. There is often some form of link between topics.

A

flight of ideas

often seen in mania but can also be seen in psychosis

How well did you know this?
1
Not at all
2
3
4
5
Perfectly
15
Q

What thought disorder is being described AND what condition often seen in?

Give the same answer to different questions

The repetition of a particular response (phrase, word, or gesture) despite the absence or cessation of the stimulus.

A

perservation

Often seen in organic brain disorders eg dementia

How well did you know this?
1
Not at all
2
3
4
5
Perfectly
16
Q

What thought disorder is being described AND what conditions often seen in?

A new, made up word that has no real meaning

A

neologism

mania and psychosis - normally as part of a delusion

can occur in autism too though

BE CAREFUL - may not be a made up word - you just might not have heard it before - always ask!

How well did you know this?
1
Not at all
2
3
4
5
Perfectly
17
Q

Define circumstantiality and tangentality and the difference between them

A

Circumstantiality is the inability to answer a question without giving excessive, unnecessary detail. However, this differs from tangentiality in that the person does eventually return the original point.

Tangentiality refers to wandering from a topic without returning to it.

How well did you know this?
1
Not at all
2
3
4
5
Perfectly
18
Q

What thought disorder is being described

when ideas are related to each other only by the fact they sound similar or rhyme

A

clang associations

How well did you know this?
1
Not at all
2
3
4
5
Perfectly
19
Q

What thought disorder is being described

completely incoherent speech where real words are strung together into nonsense sentences

A

Word salad

How well did you know this?
1
Not at all
2
3
4
5
Perfectly
20
Q

What thought disorder is being described AND what often a feature of?

there are unexpected and illogical leaps from one idea to another

A

Knight’s move thinking

severe type of loosening of associations, where there are unexpected and illogical leaps from one idea to another

a feature of schizophrenia

How well did you know this?
1
Not at all
2
3
4
5
Perfectly
21
Q

What thought disorder is being described

repetition of someone else’s speech, including the question that was asked.

A

Echolalia

How well did you know this?
1
Not at all
2
3
4
5
Perfectly
22
Q

Groups of differential diagnoses for psychosis? 5

A

1) ORGANIC
2) SUBSTANCE USE OR WITHDRAWAL
3) PSYCHOTIC CONDITIONS
4) OTHER MENTAL HEALTH CONDITIONS
5) OTHER

How well did you know this?
1
Not at all
2
3
4
5
Perfectly
23
Q

Differential diagnosis for psychosis:

  • organic disorders? 5
  • substance use or withdrawal? 4 (incl one medication)
  • psychotic conditions? 3
  • other mental health conditions? 2
  • other? 2
A

1) ORGANIC
- dementia
- delirium
- encephalitis
- post-partum
- metabolic disorders

2) SUBSTANCE USE OR WITHDRAWAL
- acute intoxication (drugs or alcohol)
- alcohol withdrawal (delirium tremens)
- cannabis-induced psychosis (or other psychosis from chronic drug use
- steroids! (also levodopa!)
(nb often get physical hallucinations with drugs)

3) PSYCHOTIC CONDITIONS
- schizophrenia
- shizo-affective disorder
- delusional disorders

4) OTHER MENTAL HEALTH CONDITIONS
- depression
- mania

5) OTHER
- sleep deprivation
- bereavement

How well did you know this?
1
Not at all
2
3
4
5
Perfectly
24
Q

definition of:

  • positive symptoms?
  • negative symptoms?

examples of negative symptoms? 7

way to remember negative symptoms?

A

POSITIVE SYMPTOMS

  • A cluster of psychotic symptoms including hallucinations and delusions.
  • A positive symptom is something added on to what most people experience (eg a hallucination)

NEGATIVE SYMPTOMS

  • A cluster of symptoms that often occur in chronic schizophrenia including poverty of speech, flat affect, poor motivation, poor attention and neglect.
  • negative symptoms are the lack of a normal experience (i.e. lacking concentration)

EXAMPLES OF NEGATIVE

  • marked apathy
  • paucity/poverty of speech
  • blunting or incongruent affect
  • social withdrawal
  • poor motivation
  • poor attention / concentration
  • neglect
THINK OF THE As
Avolition (lack of motivation)
Anhedonia
Alogia (poverty of speech)
Asociality 
Affect = blunt

negative symptoms often a late feature, less treatment responsive and have a poorer prognosis

How well did you know this?
1
Not at all
2
3
4
5
Perfectly
25
Q

SCHIZOPHRENIA

  • diagnostic criteria? (be specific)
  • incl how long symptoms go on for before give diagnosis? (what called if shorter than this?)
A
ONE OR MORE OF:
- delusion
- delusional perception
- 3rd person auditory hallucination
- thought insertion/ echo/ withdrawal/ broadcast
- delusion of control / passivity
^ie scheiner's first rank

OR

TWO OR MORE OF:

  • any other hallucination
  • negative symptoms
  • loosening of assciations
  • catatonia

lasting persistently for AT LEAST ONE MONTH (even with medication)
- AND not in acute intoxication or withdrawal OR evidence of overt brain disease (eg lewy body)

if <28 days then = acute psychotic disorder

nb often called undifferentiated psychosis for longer than a month in practice as some people recover and don’t want to give them that label

nb diagnosis of schizophrenia should not be made in the presence of extensive depressive or manic symptoms unless it is clear that schizophrenic symptoms antedated the affective disturbance

How well did you know this?
1
Not at all
2
3
4
5
Perfectly
26
Q

Definition of schizoaffective disorder?

how is it different to schizophrenia?

A

Combination of psychosis and an affective disorder (ie depression or mania)

if affective symptoms predominant then depression with psychosis, if psychosis predominant then schizophrenia - if equal impact of both AND start at the same time = schizoaffective disorder

diagnosis of schizophrenia should not be made in the presence of extensive depressive or manic symptoms unless it is clear that schizophrenic symptoms antedated the affective disturbance. If both schizophrenic and affective symptoms develop together and are evenly balanced, the diagnosis of schizoaffective disorder should be made, even if the schizophrenic symptoms by themselves would have justified the diagnosis of schizophrenia.

nb better prognosis in those with a predominant manic presentation

How well did you know this?
1
Not at all
2
3
4
5
Perfectly
27
Q

Epidemiology of Schizophrenia:

  • gender ratio?
  • peak age of onset?
  • % prevelence in UK?
A

M:F 1:1 - but men have earlier onset and often more severe
Peak onset: male = 20-28years. Female = 26-32 years

prevelence = 1%

How well did you know this?
1
Not at all
2
3
4
5
Perfectly
28
Q

Epidemiology of Schizophrenia:

  • biggest risk factor?
  • other risk factors? (2 peri-natal, 5 around life events, 2 substance misuse)
A

1) GENETIC (40% have FHx)

  • winter births
  • perinatal viral infection/trauma/hypoxia (or other pregnancy complications)
  • Social exclusion
  • Low SES
  • Childhood trauma or abuse
  • Migration
  • Urban environment
  • Cannabis (4x risk if regular use <15yo)
  • Amphetamines

nb although not causes, first episodes of psychosis are often triggered by a stressful life event (bereavement, unemployment, divorce etc) in people who are susceptible

How well did you know this?
1
Not at all
2
3
4
5
Perfectly
29
Q

Describe the dopamine theory of schizophrenia AND the 4 related brain pathways - which responsible for positive and negative symptoms and side effects of A/Ps

A

Dopamine Theory:
—> Schizophrenic patients show greater occupancy of DA receptors.
Thus, antipsychotics are all antagonists at dopamine post-synaptic receptors – mainly D2

MESOLIMBIC
= motivation, emotion, reward
- high levels in schizophrenia -> positive symptoms

MESOCORTICAL
= cognition + executive function
- low levels in schizophrenia -> negative symptoms

NIGROSTRIATAL
= stimulation of movements
- normal levels in schizophrenia (but A/Ps -> EPSEs)

TUBERINFUNDIBULAR
= dopamine acts as prolactin inhibitory hormone (PIH)
- normal levels in schizophrenia (but A/Ps -> elevated prolactin)

How well did you know this?
1
Not at all
2
3
4
5
Perfectly
30
Q

5 main types of schizophrenia?

describe each briefly

A

1) PARANOID
- eg a beautiful mind
- most common
- paranoid delusions usually with auditory hallucinations and perceptual disturbances

2) HEBEPHRENIC
- affective changes prominent (shallow + inappropriate mood)
- Irresponsible and unpredictable behaviour
- Delusions and hallucinations fleeting and fragmentary
- Thought and speech disorganised
- Tendency to social isolation
- Rapid development of negative symptoms

3) CATATONIC
- A state of immobile stupor with cardinal features such as posturing and waxy flexibility

4) SIMPLE
- Characterised by profound negative symptoms without any delusions or hallucinations.
- ie only have the negative symptoms

5) UNDIFFERENTIATED

How well did you know this?
1
Not at all
2
3
4
5
Perfectly
31
Q

what is the term for schizophrenia which intially presents over age 45?

A

paraphrenia

How well did you know this?
1
Not at all
2
3
4
5
Perfectly
32
Q

assessment of psychosis:

  • when do they need to be assessed by a psychiatrist? 2 (what’s the urgency of this?)
  • what should be done before referring to secondary care? 3
  • how is psychosis diagnosed? 2
  • what investigation can be done if suspect that psychosis is drug induced? 1
A

need to be seen by a psychiatrist

  • 1st episode psychosis
  • decline in functioning / relapse in someone being managed in primary
  • review medication to check not iatrogenic
  • take hx to check for infective signs or other signs of an organic cause
  • RISK ASSESS! (if high risk, refer same day/section)

need to see URGENTLY - espeically if initial presentation as faster you treat, better the prognosis

  • full psych Hx
  • mental state exam

urine drug screen

ALSO always think about delirium if 1st presentation and acute onset - eg sepsis etc

How well did you know this?
1
Not at all
2
3
4
5
Perfectly
33
Q

Management of psychosis/schizophrenia:

  • short-term biological? (1 + route, 1 sometimes)
  • short-term psycho-social? 2
  • long-term biological (1 always, 2 possible)
  • long-term psycho-social? 5
A

SHORT-TERM BIO

  • antipsychotics (oral/IM)
  • (sedatives)

SHORT-TERM PSYCHO-SOCIAL

  • psychoeducation (CBT/family interventions)
  • care planning

LONG-TERM BIO

  • anti-psychotic (oral/monthly depot)
  • (mood stabilisers)
  • (antidepressants)

LONG-TERM PSYCHO-SOCIAL

  • CBT for psychosis
  • art therapy
  • family interventions
  • supported employment (always ask about finances)
  • care planning

FAMILY THERAPY
especially important in families where there is high expressed emotion as there is good evidence that lots of intense emotion in the household increases the risk of relapse

COGNITIVE BEHAVIOURAL THERAPY

  • Doesn’t reduce symptoms but can help manage the distress associated with these symptoms (e.g. derogatory hallucinations)
  • Helps with an individuals understanding of their disorder

ART THERAPY
Recommended by NICE for those with negative symptoms

How well did you know this?
1
Not at all
2
3
4
5
Perfectly
34
Q

First presentation of psychosis

  • objectives in the acute management? 4
  • what team will they be under and for how long?
A

1) therapuetic trial of A/P medication
2) monitoring of mental state + response to therapy until stable
3) formulation of a care plan, allocated a care coordinator (incl a crisis plan)
4) family intervention / psychoeducation

under early intervention in psychosis team (ASPIRE in leeds) for 3 years

nb in subsequent presentations they will remain under services until they’re stable enough to be transferred back to the care of their GP

How well did you know this?
1
Not at all
2
3
4
5
Perfectly
35
Q

Prognosis following a first psychotic episode? (ie outcomes and proportion of those outcomes)

A

1/3rd recover and never have another episode

1/3rd have relapse & remitting schizophrenia but can function between relapses

1/3rd have chronic schizophrenia

How well did you know this?
1
Not at all
2
3
4
5
Perfectly
36
Q

Risk factors for relapse in psychosis/schizophrenia? 5

A
  • On-going symptoms
  • Poor compliance
  • Lack of insight
  • Substance use
  • Stopping medication suddenly

Taking an antipsychotic reduces the risk of relapse by a third

Stopping an antipsychotic within 6 months increases risk of relapse by four times

How well did you know this?
1
Not at all
2
3
4
5
Perfectly
37
Q

poor prognostic factors for psychosis / schizophrenia?

  • FHx of schizophrenia?
  • FHx of affective disorder?
  • gender?
  • marital status?
  • pre-morbid intelligence + personality?
  • age of onset?
  • speed of symptom onset?
  • presence of negative symptoms?
  • response to treatment?
  • ongoing substance use?
A
  • FHx of schizophrenia = POOR
  • FHx of affective disorder = GOOD
  • gender MALE = poor
  • marital status SINGLE = poor
  • pre-morbid intelligence + personality = good = good, SCHIZOID = poor
  • age of onset EARLY = POOR
  • speed of symptom onset INSIDIOUS = POOR
  • presence of negative symptoms = POOR
  • response to treatment, poor = poor
  • ongoing substance use = POOR
How well did you know this?
1
Not at all
2
3
4
5
Perfectly
38
Q

SCHIZO-AFFECTIVE DISORDER

- two ways to differentiate between this and psychotic depression/mania?

A

1) TIMING OF SYMPTOMS
- in schizoaffective, psychotic symptoms start at same time as change in mood (in depression/mania, affective symptoms first, then psychosis)

2) INCONGRUENT DELUSIONS
- in depression or bipolar with psychosis, the delusions are congruent with mood (eg grandiose with mania, nilhilstic with depression) - in schizoaffective they are more likely to be incongruent

How well did you know this?
1
Not at all
2
3
4
5
Perfectly
39
Q

Biological management of schizo-affective disorder? 2

A

medication norm = mood stabiliser PLUS antipsychotic

nb psycho-social management is same as schizophrenia

How well did you know this?
1
Not at all
2
3
4
5
Perfectly
40
Q

DELUSIONAL DISORDERS

  • definition? (incl features + length of symptoms)
  • what type of delusions not compatible with?
  • gender more common in?
A

Delusions are the most prominent feature

  • typically well formed and long standing (they can be life-long!)
  • Hallucinations aren’t present or are only fleeting
  • no marked blunting of affect.
  • Symptoms >1 month.

Delusions of control/passivity experiences aren’t compatible with this diagnosis

more common in females.

How well did you know this?
1
Not at all
2
3
4
5
Perfectly
41
Q

sub-types of delusional disorder:

- describe PERSECUTORY

A

Belief that they, or someone close to them is being mistreated, or that someone is planning to harm them.

How well did you know this?
1
Not at all
2
3
4
5
Perfectly
42
Q

sub-types of delusional disorder:

- describe OTHELLO SYNDROME

A
  • Morbid jealousy
  • Delusional belief that partner is having affair
  • No evidence/ misinterprets minor evidence
  • Association with alcohol dependence/ sexual dysfunction
  • Risk of stalking/ violence

“always get jealous partners in shakespeare plays”

How well did you know this?
1
Not at all
2
3
4
5
Perfectly
43
Q

sub-types of delusional disorder:

- describe DE CLERAMBAULT’S SYNDROME (what aka?)

A
  • Aka erotomania

Delusional belief a famous person/ of higher social status, is in love with them.
- Cannot declare love, so communicate via secret signs.

“CLERAM = like CLAMBER - you clamber over all the other fans to get to the celebrity”

How well did you know this?
1
Not at all
2
3
4
5
Perfectly
44
Q

sub-types of delusional disorder:

- describe CAPGRAS SYNDROME

A

Delusional misidentification

Belief that relative/ spouse/ close friend has been replaced by identical looking double or impostor

“someone has taken loved one’s CAP and is trying to interpret them”

How well did you know this?
1
Not at all
2
3
4
5
Perfectly
45
Q

sub-types of delusional disorder:

- describe FREGOLI’S SYNDROME

A

Different people are a single person who changes appearance or is in disguise.

Often believe they are being persecuted by that person

“can’t be FREE of the person who is shape-shifting to continue persecuting you”

How well did you know this?
1
Not at all
2
3
4
5
Perfectly
46
Q

sub-types of delusional disorder:

- describe COTARD’S SYNDROME

A

A nihilistic delusion without other psychotic symptoms.

Delusion of being dead/ dying/ non-existent/ rotting

patient believes that they (or in some cases just a part of their body) is either dead or non-existent. This delusion is often difficult to treat and can result in significant problems due to patients stopping eating or drinking as they deem it not necessary.

Associated with severe depression and psychotic disorders

“COT-death - believe they are dead or dying”

How well did you know this?
1
Not at all
2
3
4
5
Perfectly
47
Q

sub-types of delusional disorder:

  • describe EKBOM’S SYNDROME
  • what associated with?
A

Delusional Parasitosis – infested with parasites

No evidence – only small marks/ freckles

Risk of self-harm through excoriation to get rid of parasites.

Often present to Derm.

Associated with Cocaine.

“tiny BOMBS of insects under their skin”

How well did you know this?
1
Not at all
2
3
4
5
Perfectly
48
Q

sub-types of delusional disorder:

  • describe FOLIE A DEUX
  • aka?
A

aka Induced delusional disorder – psychosis shared by two people.

The dominant partner has a delusional belief and then the dependent partner develops the same belief.

The dominant partner requires treatment but the dependent partner recovers upon separation.

“DEUX = two, delusion shared by 2 people”

How well did you know this?
1
Not at all
2
3
4
5
Perfectly
49
Q

DELUSIONAL DISORDER:

  • biological management?
  • psycho-social management?
  • prognosis?
A

anti-psychotics

psych-social = same as schizophrenia

Is particularly difficult to treat - even with intensive treatment

People often retain some aspect of the delusion or continue to hold the belief but are less preoccupied with it

How well did you know this?
1
Not at all
2
3
4
5
Perfectly
50
Q

Acute psychotic disorder:

- definition?

A

person has psychosis for LESS THAN 28 DAYS

ie too short in duration to diagnosis schizophrenia, delusional disorder, schizoaffective disorder

How well did you know this?
1
Not at all
2
3
4
5
Perfectly
51
Q

Depression:

  • porportion of people who will have an episode in their lifetime?
  • gender more common in?
A

1/5th

more common in female (2:1)

How well did you know this?
1
Not at all
2
3
4
5
Perfectly
52
Q

DDx for depression:

  • psych? 10
  • if been present for ‘all their life’? 2
  • organic? 8

what should always be reviewed before giving a diagnosis of depression?

A

PSYCH

  • seasonal affective disorder
  • adjustment disorder (eg grief)
  • drug-induced affective disorders (eg alcohol, cannabis)
  • dementia (incl parkinsons)
  • anxiety (may be co-morbid)
  • OCD
  • eating disorders
  • bipolar
  • schizo-affective
  • schiophrenia (esp if negative symptoms)

ALL LIFE

  • dysthymia
  • emotionally unstable personality disorder

ORGANIC

  • hypothyroidism
  • cushings (incl med-induced)
  • anaemia
  • cancer
  • IBD
  • EBV
  • chronic fatigue (>6mo)
  • MS (or stroke or other CNS disorder)

ALWAYS REVIEW MEDICATION LIST (as some medications can mimic depression)

How well did you know this?
1
Not at all
2
3
4
5
Perfectly
53
Q

Common groups of medications that can mimic / cause depression? 7

A
  • anti-psychotics
  • anti-convulsants (incl gabapentin)
  • beta-blockers
  • PPIs + H2 blockers
  • corticosteroids
  • contraceptives
  • retinoids (for severe acne)
How well did you know this?
1
Not at all
2
3
4
5
Perfectly
54
Q

What proportion of pts with depression will also meet criteria for another psychiatric disorder?

  • what other co-morbid psych conditions should be screened for when doing a psych hx? 4 (incl example of how would ask about each)
A

2/3rds

1) MANIA
- ‘have you ever had a period when you’ve had a lot of energy and managed to do a lot of things, despite having very little sleep?’

2) PSYCHOSIS
- ‘sometimes when people are feeling really low they can hear voices or see things that others can’t hear or see - has that ever happened to you? do you have any beliefs that you don’t think are shared by others?’

3) ANXIETY
- ‘many people who feel low in mood also experience feelings of anxiety and feel anxious, nervous or on edge about things - do you feel like this? have you ever had any panic attacks?’ (get them to describe the latter)

4) ALCOHOL/SUBSTANCE MISUSE
- ‘sometimes when people are feeling very low, they find that they start drinking more alcohol or using other medications to escape from those feelings, is that something you experience? how much alcohol do you drink? do you use any recreational drugs?’

How well did you know this?
1
Not at all
2
3
4
5
Perfectly
55
Q

RISK FACTORS FOR DEPRESSION

  • biggest risk factor for an episode of depression?
  • bio? 5
  • psycho? 3
  • social? 7
A

BIGGEST = previous depressive episode

BIO

  • genetic / FHx
  • female
  • chronic disease eg stroke (esp ones causing disability or pain)
  • post-partum
  • heavy alcohol or illicit substance use

PSYCHO

  • adverse childhood experience (parental bereavement, neglect, parental alcoholism, childhood sexual abuse)
  • personality traits (anxiety, obsessive, impulsive)
  • low self-esteem

SOCIAL

  • unemployment
  • low socio-economic status
  • seperation or divorce
  • bereavement (seperate from normal adjustment reaction)
  • adverse life event
  • migration
  • LGBT+
How well did you know this?
1
Not at all
2
3
4
5
Perfectly
56
Q

What are the 4 Ps for causes of a psychiatric illness

A

BIO, PSYCH and SOCIAL for:

1) PREDISPOSING
2) PRECIPITATING
3) PERPETUATING
4) PROTECTIVE

How well did you know this?
1
Not at all
2
3
4
5
Perfectly
57
Q

SYMPTOMS OF DEPRESSION:

  • core? 3
  • psychological? 10
  • biological / somatic? 6
  • social? 3

duration of symptoms to make a diagnosis?

A

CORE

1) anhedonia
2) low mood (often diurnal variation)
3) low energy / fatigue

PSYCHOLOGICAL

  • worthlessness
  • low self-esteem
  • tearful
  • guilt
  • irritable / intolerant of other
  • low motivation
  • struggle to make decisions
  • poor concentration
  • hopelessness + helplessness
  • suicidal thoughts

BIOLOGICAL

  • Change in APPETITE (decrease is typical, increase is atypical)
  • SLEEP disturbance (early morning waking is typical - but can get hypersomnia = atypical)
  • low SEX drive
  • PSYCHOMOTOR retardation (moving or speaking slowly)
  • CONSTIPATION
  • ACHES and PAINS

SOCIAL

  • avoiding friends
  • neglecting hobbies/interests
  • having difficulties in home, work or family life

symptoms must be present for at least TWO WEEKS
- (not secondary to effects of drug/ alcohol misuse, organic illness, or bereavement (before 6 months)

nb pt w bipolar more likely to have atypical sleep and apetite compared to unipolar depression

How well did you know this?
1
Not at all
2
3
4
5
Perfectly
58
Q

How assess severity of depression?

  • what additional group of symptoms can occur in severe depression?
A

mild depression – has some impact on your daily life

moderate depression – has a significant impact on your daily life

severe depression – makes it almost impossible to get through daily life; a few people with severe depression may have psychotic symptoms

nb can do based on symptom count, PHQ-9 etc - but in reality do based on level of functioning

How well did you know this?
1
Not at all
2
3
4
5
Perfectly
59
Q

Two key differentiators between grief and depression?

A

People who are grieving find their feelings of sadness and loss come and go, but they’re still able to enjoy things and look forward to the future.

In contrast, people who are depressed constantly feel sad. They find it difficult to enjoy anything or be positive about the future

How well did you know this?
1
Not at all
2
3
4
5
Perfectly
60
Q

Patients over 55 may present differently with depression - some symptoms / features that are more likely in this age group? 6

A
  • Psychomotor retardation
  • Pseudodementia
  • Hypochondriacal overvalued/delusions
  • Obsessive-compulsive symptoms
  • Agitated depression
  • Apathy
How well did you know this?
1
Not at all
2
3
4
5
Perfectly
61
Q

PSEUDODEMENTIA:

  • what is it?
  • how differ from dementia?
A

depression in older people (ie depression can often present very similar to dementia)

symptoms incl:

  • memory loss
  • imparied executive functioning

ONSET

  • fast in pseudodementia
  • insidious in dementia

MEMORY LOSS

  • global in pseudodementia
  • short-term initially in dementia

COGNITIVE TESTS

  • say ‘i don’t know’ in peudodementia
  • try to answer Qs but fail in dementia

depressed mood in pseudodementia, incongruity of affect in dementia

How well did you know this?
1
Not at all
2
3
4
5
Perfectly
62
Q

How to screen for 3 core symptoms of depression? (ie what Qs to ask)

A

MOOD
‘During the last month, have you often been bothered by feeling down, depressed or hopeless?’

ANHEDONIA
‘During the last month, have you often been bothered by having little interest or pleasure in doing things?’

FATIGUE
‘during the last month, have you often felt very fatigued or that you had no energy?’

How well did you know this?
1
Not at all
2
3
4
5
Perfectly
63
Q

INVESTIGATION OF LOW MOOD / SUSPECTED DEPRESSION

  • hx + exam to do? (what assessment should this include)
  • bedside investigation? 1
  • other investigations to consider to rule out organic causes?
A
  • full psych hx (and systems review)
  • including FULL risk assessment (to self, to others incl safeguarding, from others)
  • MSE
  • PHQ-9

if suspect organic cause:

  • physical exam
  • review meds
  • bloods (esp TFTs + FBC)
  • (imaging if neuro signs)
How well did you know this?
1
Not at all
2
3
4
5
Perfectly
64
Q

possible effects of despression on physical health? 4

what % of depression in general hospital patients goes undetected?

A
  • pain can be harder to control
  • may have less motivation to engage in treatment or take medication
  • self-neglect (incl not attending appts etc)
  • doubles risk of getting type 2 DM

50% of depression in general hospital pts goes undetected

How well did you know this?
1
Not at all
2
3
4
5
Perfectly
65
Q

NICE management of depression:

  • lifestyle changes for all? 3
  • bio/psycho for mild-moderate depression? 1
  • bio/psycho for moderate-severe? 2
  • what other aspect of management?
  • where to look for how to manage in primary care?
A

LIFESTYLE

  • reduce alcohol intake
  • sleep hygiene
  • regular exercise

MILD-MODERATE
- low-intensity psychological interventions (eg via IAPT)
(ie not medication)

MODERATE-SEVERE

  • high-intensity psychological intervention
  • antidepressant (SSRI 1st line)

plus SOCIAL management (see other flashcard)

NICE CKS on initial management of depression is very good! - incl leaflets etc

How well did you know this?
1
Not at all
2
3
4
5
Perfectly
66
Q

Areas to focus on to help social management of depression? 6

A

OTHER MEDICAL CONDITIONS
- get these under control, incl pain etc

ADLs

  • ‘take me through a typical day’
  • address holes eg older people may need additional support
  • also CARERs need support too!

HOUSING

  • isolated?
  • issues?

EMPLOYMENT

  • working? problems at work?
  • can provide letters/reports of support

FINANCES

  • any debts?
  • citizen’s advice re debt management plans
  • referral to food banks

ACTIVITIES

  • if lonely/isolated/bored
  • gardening? social group?
  • social prescribing!

“if easier to remember can go through ASD OHA DOT”

How well did you know this?
1
Not at all
2
3
4
5
Perfectly
67
Q

Examples of low-intensity psychological support for depression? 4

A
  • individual guided self-help (norm CBT-based)
  • CCBT
  • group-based CBT
  • structured group-based physical activity programme
How well did you know this?
1
Not at all
2
3
4
5
Perfectly
68
Q

examples of high-intensity psychological support for depression? 4

A
  • individual CBT
  • interpersonal therapy (IPT)
  • behavioural activation
  • couple therapy (although not commonly used)

nb each of the above norm 16-20 sessions over 3-4 months

Counselling and short-term psychodynamic therapy — these can be considered for people who decline antidepressants and high-intensity psychological interventions
- but counsel patients on the uncertainty of effectiveness of these interventions

How well did you know this?
1
Not at all
2
3
4
5
Perfectly
69
Q

How to explain CBT to a patient

A
  • type of talking therapy
  • structured and practical
  • involves problem and goal definition
  • use diaries to monitor and identify problematic thoughts, or other problems and monitoring progress
  • help you learn to think in more helpful ways, includes behavioural experiments
  • based on the here and now (not exploring past)
  • based on the cognitive model (how we think affects how we feel, behave and our biology)

nb see my notes for more infor on CBT

How well did you know this?
1
Not at all
2
3
4
5
Perfectly
70
Q

How to describe interpersonal therapy (IPT) to a patient?

A

structured talking therapy that typically works intensely on established interpersonal issues. The underlying belief of interpersonal therapy is that psychological symptoms (such as depression) are often a response to difficulties we have interacting with others. The resulting symptoms can then also affect the quality of these interactions, causing a cycle. The thought process behind the therapy is that once a person is capable of interacting more effectively with those around them, the psychological symptoms can improve.

IPT focusses on:

  • conflicts
  • life changes
  • grief and loss
  • relationship problems
71
Q

How to describe behavioural activation to a patient?

A

Focuses on addressing behaviour:

  • When we are depressed we stop doing the things that usually give us pleasure which makes us even more depressed.
  • We isolate ourselves and avoid interactions with others as this might be difficult e.g. going back to work after time off sick
  • We wait until we feel better before doing things - unfortunately we never feel better so never do things

Treatment involves getting people to act according to a plan rather than how they feel and involves doing things!
- Simple goals to start e.g. walk dog every day and build up

72
Q

Indications for ECT? 4

describe what happens

A
  • life-threatening depression
  • moderate-severe depression that has responded to ECT in the past
  • prolonged or severe mania
  • catatonia

2 x weekly treatment - approx 12 in total

unilateral vs bilateral

short GA and muscle relaxant - then short induced controlled seizure

nb 70-80% response in depression

73
Q

Side effects of ECT? 4

A
  • short-term memory loss
  • headache
  • nausea
  • muscle pain
74
Q

Prescribing anti-depressants:

  • expectations to tell patients? 5
  • when to follow up?
  • how long to continue for? why? how to stop?
A
  • try for at least 4-6 weeks
  • effects will take a few weeks and energy improve first, then mood (also get SEs before therapeutic effect)
  • warn about suicide risk (give crisis numbers)
  • different ones work for different people, try this one first and can try another if this doesn’t work
  • not addictive but don’t stop suddenly as may get discontinuation symptoms

FOLLOW UP after 2 weeks, 1 week if under 30 (as higher suicide risk)

continue for 6-9 months AFTER resolution of symptoms

  • to reduce risk of relapse
  • reduce dose gradually (to avoid discontinuation symptoms)

nb if multiple episodes, consider for at least 2 years

75
Q

Possible discontinuation symptoms with antidepressants? 7

What % of patients experience these?

A
  • restlessness
  • problems sleeping
  • unsteadiness
  • sweating
  • abdominal symptoms
  • altered sensations (for example electric shock sensations in the head)
  • altered feelings (irritability, anxiety, confusion)

1/3rd pt experience these when coming off treatment

76
Q

If antidepressants aren’t working after 4-6 weeks:

  • what should be reviewed / checked? 6
  • what to do/consider if still not working and all the above have been addressed? 4

nb this is assuming they are already having psychological therapy

A

REVIEW

  • Review DIAGNOSIS
  • Check COMPLIANCE
    (they may not but be too embarrassed to say that they forget)
  • Check taking correct DOSE
  • Review ALCOHOL intake
    (will stop antidepressants being effective)
  • Check for OTHER DRUGS
  • Address PSYCHOLOGICAL and SOCIAL factors

ONLY THEN:

1) INCREASE dose
2) SWITCH drugs
3) AUGMENT with lithium or A/Ps
4) ECT if severe and not responsive

77
Q

When to refer someone with depression to secondry care:

  • as an emergency? 4
  • for routine? 3
A

EMERGENCY

1) significant risk of self-harm
2) danger to others
3) psychotic symptoms
4) severe agitation

ROUTINE

1) significant depression with functional impairment persisting despite adequate treatment in primary care
2) when additional community support from community mental health team or day hospital staff is required
3) indication for specialist psychological treatment (eg ECT, lithium)

78
Q

Prognosis for depression:

  • average length of episode?
  • chance of recurrence after initial episode?
  • what % have a manic episode?
A

—> Average length of depressive episode = 6 months

  • after a first episode of depression, 50% chance of having another one

chance of recurrence increases if have second episode

Median is 4 episodes in lifetime

10 % go on to have manic episode

79
Q

SYMPTOMS OF MANIA

  • possible moods? 3
  • activity? 7
  • cognition/thoughts? 2
  • additional signs on MSE? 2
A

MOOD =

  • abnormally ELEVATED
  • extreme IRRITABILITY
  • sometimes AGGRESSION

ACTIVITY

  • INCREASED energy / activity
  • RESTLESSNESS
  • decreased need for SLEEP
  • DISTRACTIBILITY / poor concentration
  • IMPULSIVITY
  • increased LIBIDO / sexual indiscretions
  • DISINHIBITION

THOUGHTS

  • extravagant or impractical PLANS (eg business investments, spending sprees)
  • PSYCHOTIC symptoms (norm grandiose delusions, auditory hallucinations) - these are CONGRUENT with mood

nb if ingongruent with mood then suspect schizoaffective

MSE (all of the above plus:)

  • pressured or incomprihensible speech
  • flight of ideas or racing thoughts
80
Q

Three main differences between mania + hypomania?

A

MANIA

  • at least 7 days
  • may have psychotic symptoms
  • severe enough to cause marked impairment in social or occupational functioning
  • may need hospitalisation

HYPOMANIA

  • at least 4 days
  • no psychotic symptoms
  • not severe enough to cause marked impairment in social or occupational functioning
  • no hospitalisation

so as soon as an episode has ANY of the below things, it is, by default a manic episode:

  • lasts 7 days
  • psychotic symptoms
  • has severe impact on functioning
  • requires hospitalisation
81
Q

What is a ‘mixed episode’ in bipolar disorder?

A

A mixed episode is:

1) A mixture or rapid alternation of manic and depressive symptoms, or
2) A period of time (at least 1 week) in which the criteria are met for either a manic or hypomanic episode and at least three symptoms of depression are present during the majority of the days of the current or most recent episode of mania or hypomania, or
3) A period of time (at least 2 weeks) in which the criteria for a major depressive episode are met and at least three manic or hypomanic symptoms are present during the majority of days of the current or most recent episode of depression.
basically: what it suggests - is a mix of manic and depressive symptoms

82
Q

Give an example of what level of symptoms someone with a hypomanic episode would have?

A
  • Mild elevation of mood, or irritability.
  • Increased energy and activity, which may lead to increased performance at work or socially.
  • Feelings of well-being, or physical and mental efficiency.
  • Increased sociability, talkativeness, and over-familiarity.
83
Q

BIPOLAR AFFECTIVE DISORDER (BAD)

  • definition?
  • what are the 4 types of episodes? (how long should each last to ‘count’ as an episode)
  • % who start with a depressive episode?
A

Mood disorders in which pts experience bouts of mania, hypomania, depression and mixed states
- may be long periods (years) of recovery between

DEFINITION:

  • after at least 2 episodes of disturbed mood
  • one of which should be manic or hypomanic

NB usually complete recovery between episodes

  • depression (2 weeks)
  • mania (7 days)
  • hypomania (4 days)
  • mixed (7-14 days - depends)
  • 50% start with depressive episode - don’t give them diagnosis of bipolar until have a manic episode (always screen for past mania if depression)

nb in DSM-5 have type 1 and 2 (2 is hypomanic, don’t need manic episodes - type 1 must have manic episodes - this is the main difference between them)
- but in ICD-10 (use in UK) don’t distinguish between type 1 and 2

84
Q

BIPOLAR AFFECTIVE DISORDER:

  • prevelence?
  • gender difference?
  • mean age onset?
A

1% prevelence

M=F

21 mean age of onset

“both men & women go on a spending spree for their 21st”

85
Q

BIPOLAR AFFECTIVE DISORDER:

  • biological etiology theory?
  • biggest risk factor?
  • common triggers of initial presentation? 2
A

MONOAMINE THEORY
- mania results from INCREASED levels of noradrenaline, serotonin + dopamine

GENETICS / FHx = biggest risk factor
(largest heritability of any

COMMON INITIAL TRIGGERS

  • severe stress / disruption
  • drugs (eg cocaine, amphetamines)
86
Q

what things someone presenting with depression make you suspect this might be bipolar? 3

A

features of depression that make bipolar more likely

  • FHx of bipolar
  • hypersomina
  • hyperphagia
87
Q

Differences between bipolar and unipolar depression:

  • FHx
  • Hx of mania
  • sex ratio
  • age of onset
  • speed of onset
  • psychosis
  • activity
  • sleep
A

FHx of BAD

  • BD = yes
  • UD = no

Hx of Mania

  • BD = yes
  • UD = no

sex ratio

  • BD = M=F
  • UD = F>M

Age of onset

  • BD = early
  • UD = 30-50 (can be earlier)

Onset

  • BD = abrupt
  • UD = insidious

Psychosis

  • BD = more common
  • UD = less common

activity

  • BD = retardation
  • UD = agitation

sleeo
- BD = hypersomnia
- UD = insomnia
(same w appetite)

88
Q

DDx for manic symptoms:

  • psych? 4
  • medications/substances? 4
  • other organic? 8
A

PSYCH

  • BAD
  • schizoaffective disorder
  • schizophrenia
  • ADHD

MEDS/SUBSTANCES

  • stimulants
  • alcohol
  • steroids
  • dopamine agonists

ORGANIC

  • hyperthyroidism
  • cushings
  • epilepsy (ask re focal features)
  • frontal lobe disease
  • multiple sclerosis
  • huntington’s disease
  • HIV
  • sleep deprivation
89
Q

Presentations of mania:

  • hx and exam to do?
  • assessment to always do?
  • bedside investigations? 2
  • bloods to do? (4 always, 1 if already on treatment)
  • other investigation to consider?
A
  • full psych hx
  • MSE
  • ECG
  • Urine drug screen

BLOODS

  • FBC
  • U+E
  • LFT
  • Calcium (high or low can trigger)
  • LITHIUM LEVELS (if on, to assess compliance)

consider CT head if first presentation or any focal or abnormal signs

90
Q

Acute management of MANIA

  • biological options? 4
  • psycho/social? 3
A

BIOLOGICAL
- atypical antipsychotic (if psychotic)
OR
- benzos

to manage initial hyperactivity

then TITRATE up lithium (or other mood stabiliser)

consider short-term hypnotics (z drugs) if sleep deprived

1) ADMIT pt while tirating up mood stabiliser for own safety
2) risk assessment before discharge
3) stop driving (if relevant) - also consider restricting access to finances

91
Q

LONG TERM MANAGEMENT of BIPOLAR AFFECTIVE DISORDER:

  • biological (1 norm, 1 sometimes) - incl length of this
  • psychological? 2
  • social? 3
  • management for intractible mania? 1
A

BIOLOGICAL

  • mood stabilisers, lithium 1st line
  • (antipsychotics)

length of treatment depends on sevrity and frequency of episodes, need regular follow up to manage, can consider weaning off if been symptom-free for mnay years but this may provoke a relapse

nb can use antidepressants in some pts but tend to avoid as can trigger mania/hypomania

PSYCHOLOGICAL

  • psycho education to pt + family (incl compliance aids, recognising triggers + symptoms etc)
  • BAD-specific psychotherapies (eg CBTetc)

SOCIAL

  • crisis management plan / care coordinator
  • avoid recreational drugs
  • address financial, employmert etc issues

ECT for intractible / severe mania (but rarely used)

92
Q

Risks in BIPOLAR AFFECTIVE DISORDER (esp Mania)

  • most important risks to self? 2
  • other risks to self? 6
  • risk to others? 2
  • risk from others? 1
A

TO SELF
= self-harm
= suicide

  • financial difficulties (overspending)
  • damage to reputation/occupation/relationships
  • traumatic injuries/accidents
  • STIs/unplanned pregnancy
  • self-neglect / exhaustion / dehydration
  • alcohol and substance misuse

TO OTHERS

  • reckless driving / injuries
  • neglect of children/dependants

FROM OTHERS
- exploitation (dt lack of insight)

nb much higher risk of suicide than in unipolar depression! - risk is highest in post-manic depressive phase

nb hypomanic patients are at risk too but this often goes unnoticed

life charts and symptom diaries may also be helpful for some people

93
Q

NEUROTIC, STRESS-RELATED AND SOMATIFORM DISORDERS:

  • phobic anxiety disorders? 3
  • other anxiety disorders? 2
  • other? 1
  • reactions to severe stress? 3
  • examples of features of dissociative / conversion disorders? 7
  • somatoform disorders? 2
A

PHOBIC ANXIETY

  • agoraphobia
  • social phobia
  • specific phobias

OTHER ANXIETY

  • panic disorder
  • generalised anxiety disorder

OTHER
- obsessive compulsive disorder

REACTIONS TO SEVERE STRESS

  • acute stress reaction
  • post-traumatic stress disorder
  • adjustment disorders
DISSOCIATIVE / CONVERSION DISORDERS
- Amnesia
- fugue
- stupor
- motor
- convulsions
- trance &amp; possession states
- anaesthesia and sensory loss 
(this include depersonalisation, derealisation, conversion disorders)

SOMATOFORM DISORDERS

  • somatisation disorder
  • hypochondrial disorder
94
Q

Generalised anxiety disorder (GAD)

  • definition? (incl length of time)
  • 3 groups of features?
A

The essential feature is anxiety, which is generalized and persistent but not restricted to, or even strongly predominating in, any particular environmental circumstances (i.e. it is
“free-floating”).

As in other anxiety disorders the dominant symptoms are highly variable, but complaints of continuous feelings of nervousness, trembling, muscular tension, sweating,
lightheadedness, palpitations, dizziness, and epigastric discomfort are common.

Fears that the sufferer or a relative will shortly become ill or have an accident are often expressed, together with a variety of other worries and forebodings.

ICD-10: The sufferer must have primary symptoms of anxiety MOST DAYS for at least SEVERAL WEEKS at a time, and usually for several months (DSM IV says 6 mo).

These symptoms should usually involve elements of:

1) APPREHENSION (worries about future misfortunes, feeling “on edge”, difficulty in concentrating, etc.);
2) MOTOR TENSION (restless fidgeting, tension headaches, trembling, inability to relax); and

3) AUTONOMIC OVERACTIVITY (lightheadedness, sweating, tachycardia or tachypnoea, epigastric
discomfort, dizziness, dry mouth, etc.).

could also split into 4 groups:

  • psychological
  • arousal
  • motor
  • autonomic
95
Q

psychological symptoms of GAD? 4

A
  • Worry/ Apprehension/ Fear
  • Persistent nervousness
  • Poor Concentration
  • Irritability
96
Q

arousal symptoms of GAD? 4

A
  • hypervigilance
  • increased startle response
  • restlessness
  • insomnia (struggle geting to sleep)
97
Q

motor tension symptoms of GAD?

nb these don’t include autonomic symptoms

A
  • musle tension
  • headaches
  • trembling
  • purposeless activity
98
Q

autonomic symptoms of GAD:

  • CVS? 3
  • resp? 2
  • GI? 6
  • urinary tract? 1
  • neuro? 3
  • other? 3
A

CVS

  • palpitations
  • chest tightness
  • chest pain

RESP

  • difficulty inhaling
  • hyperventilation

GI

  • dry mouth
  • loss of appetite (or craving food)
  • nausea
  • ‘butterflies’
  • epigastric pain
  • loose stools

URINARY
- frequent micturation

NEURO

  • blurred vision
  • light headedness
  • pins and needle

OTHER

  • sweating
  • depersonalisation
  • derealisation
99
Q

EPIDEMIOLOGY OF GAD

  • which gender more common in?
  • predisposing risk factors? 4
  • precipitating risk factors? 2
  • what patient to suspect GAD in who dosn’t actually present with anxiety?
A

more common in female

PREDISPOSING

  • FHx of anxiety
  • Hx of physical or emotional trauma
  • Hx of other anxiety disorders (panic, social phobia, specific phobias)
  • Hx of substance abuse

PRECIPITATING

  • current physical or emotional trauma
  • Chronic pain or physical illness (such as arthritis, cancer, coronary heart disease, cerebrovascular accident, or COPD).

Have high level of suspision for GAD in patients who make repeated visits with the same physical symptoms which do not respond to treatment (for example insomnia, headache, or fatigue)

100
Q

What psychological conditions are often co-morbid with GAD? 8

A
  • depression
  • panic disorder
  • agoraphobia
  • social phobia
  • specific phobias
  • health anxiety
  • OCD
  • alcohol (and other substance, incl cannabis) misuse
101
Q

What medications / substances can mimic / exacerbate GAD:

  • prescribed? 5
  • not-prescribed? 4
A

PRESCRIBED

  • salbutamol
  • theophylline
  • beta-blockers
  • steroids
  • some antidepressants (esp initially)

NOT-PRESCRIBED

  • alcohol use or withdrawal
  • other illicit substances (use or withdrawal)
  • caffeine
  • herbal medicines (including ma huang, St. John’s wort, ginseng, guarana, belladonna)
102
Q

Assessment for suspected GAD:

  • history and 2 exams to do?
  • possible findings on physical exam? 4
  • bedside tool to assess severity? 1
  • what else to assess? 1
A
  • psych hx
  • MSE
  • physical exam (to exclude DDx)

possible findings on physical exam:

  • increased HR
  • SOB
  • trembling
  • exaggerated startle response

GAD-7 questionnaire

ALWAYS risk assess (to self, to others, from others)

103
Q

DDx for GAD
- psych?

(see other flashcard for organic DDx)

A

PSYCH

  • SITUATIONAL anxiety (controllable anxiety that is not generally associated with pathological symptoms and relates to a particular life event, such as an upcoming exam)
  • ADJUSTMENT disorder (temporary anxiety that has occurred in response to a life stressor and persists for no longer than 6 months after the stressor ends)
  • DEPRESSION (can be co-morbid)
  • PANIC disorder (recurrent episodes of sudden onset anxiety, in the absence of multi-themed worry - can be co-morbid)
  • SOCIAL phobia (anxiety or persistent fear that is limited to social situations and fear of social scrutiny or embarrassment. Avoidance behaviour is commonly present.)
  • OCD (anxiety due to compulsions or obsessions)
  • PTSD (anxiety that is caused by exposure to reminders of past trauma)
  • SOMATOFORM disorders (anxiety related to specific physical complaints, which have no medical basis)
  • ANOREXIA NERVOSA (axiety related to fear of gaining weight)
  • SUBSTANCE and ALCOHOL misuse / withdrawal (frequently associated with anxiety, especially among men)
  • MEDICATION-INDUCED anxiety (anxiety triggered by exposure to medications including salbutamol, theophylline, corticosteroids, antidepressants, and some herbal medicines. Caffeine may also be a trigger. Withdrawal from CNS depressants can also trigger anxiety)
104
Q

ORGANIC DDx for GAD? 7

incl how to exclude on symptoms / physical exam

(see other flashcard for psych DDx)

A

CARDIAC DISEASE

  • symptoms such as palpitations, sensation of rapid or skipped heartbeat, dizziness, dyspnoea and chest pain on exertion, and numbness
  • Examination may reveal hypertension, hypotension, tachycardia or bradycardia, or S3 or S4 gallops

RESP DISEASE

  • suggested by a hx of lung conditions such as asthma or COPD, the person being a current or previous smoker, and signs/symptoms such as wheezing, cough, respiratory distress, or sputum production, and a feeling of suffocation.
  • Pulse oximetry may reveal low oxygen sats

HYPERTHYROIDISM
- weight loss, warm moist skin, heat intolerance, ophthalmopathy, or goitre

ANAEMIA
- fatigue, dyspnoea, and palpitations

INFECTION
- anxiety and additional symptoms of fever, night sweats, or cough, which resolve following successful treatment

IRRITABLE BOWEL
- alteration of bowel habit associated with pain, and abdominal discomfort, bloating, or distention

PHAEOCHROMOCYTOMA
- anxiety that co-exists with hypertension and/or tachycardia

105
Q

GAD

  • bio management options? 2
  • psych management? 1
  • social management? 4

which should you do first?

what to monitor for with bio treatment? 2

A

BIO

  • SSRIs
  • (benzos - short term only!)

PSYCH
- CBT (group, CCBT, 1-to-1)

SOCIAL

  • address substance misuse
  • regular exercise
  • sleep hygiene
  • practically address stressors (financial, relationships, work)

(also make sure any co-morbid medical conditions are managed)

do:

  • 1) social
  • 2) psych
  • 3) meds

(although can do meds + psych simultaneously)

nb people w GAD are particularly prone to ADR - so tend to start with lower doses and rise slower than, eg for depression
- expect result in 6-12 wks

monitor for:

  • increased anxiety or akasthesia
  • suicidal ideation

NICE suggest a step-wise approach:
step 1: education about GAD + active monitoring
step 2: low intensity psychological interventions (individual non-facilitated self-help or individual guided self-help or psychoeducational groups)
step 3: high intensity psychological interventions (cognitive behavioural therapy or applied relaxation) or drug treatment (sertraline 1st line)
step 4: highly specialist input e.g. Multi agency teams

106
Q

Describe a panic attack

A

PANIC ATTACK = discrete episode of intense fear or discomfort.

which are not restricted to any particular situation or set of circumstances, and which are therefore unpredictable

Starts abruptly, reaches maximum with few minutes.

Features at least 4 specific symptoms of anxiety.

dominant symptoms vary from person to person, but sudden onset of palpitations, chest pain, choking sensations, dizziness, and feelings of unreality
(depersonalization or derealization) are common.

There is also, almost invariably, a secondary fear of dying, losing control, or going mad.

frequent and unpredictable panic attacks produce fear of being alone or going into public places. A panic attack is often followed by a persistent fear of having another attack.

nb one panic attack doesn’t mean you have a disorder

107
Q

PANIC DISORDER

- definition? (incl time frame)

A

several (norm 4) severe attacks of autonomic anxiety should have occurred within a
period of about 1 month:

(a) in circumstances where there is no objective danger;
(b) without being confined to known or predictable situations; and
(c) with comparative freedom from anxiety symptoms between attacks (although anticipatory anxiety is common).

ie differentiate from GAD and specific phobias

Panic disorder should not be given as main diagnosis if the patient has depressive disorder at the time the attacks start (panic attacks are probably secondary to depression)

can occur with or without agoraphobia

108
Q

PANIC DISORDER

- risk factors? 5

A
  • FHx of panic or other anxiety disorder
  • stress
  • life changes / traumatic event
  • smoking
  • excess caffeine
109
Q

PANIC DISORDER

  • organic DDx of a panic attack that are potentially life threatening? 5
  • other groups of organic conditions which could be mistaken for panic disorder?
  • psych DDx? 5
A

LIFE THREATENING

  • anaphylaxis (or other airway obstruction)
  • asthma attack
  • PE
  • ACS
  • arrythmia / SVT

OTHER ORGANIC

  • CARDIOvascular diseases (incl angina, heart failure)
  • PULMonary diseases (incl asthma)
  • NEUROlogical diseases (incl epilepsy, TIA, huntingtons)
  • ENDOCRINE diseases (addisons, thyroid, menopause, hypoglycaemia)
  • drug INTOXICATION (amphetamines, hallucinogens, cocaine)
  • drug WITHDRAWAL (alcohol, opiates, sedatives)

PSYCH

  • GAD
  • agoraphobia
  • specific phobia
  • PTSD
  • depression
110
Q

PANIC DISORDER assessment:

  • history and 2 exams?
  • what trying to identify during hx?
A
  • psych hx
  • MSE
  • physical exam (esp heart and lungs)

try and identify any triggers (may lead to specific phobia instead of panic disorder)

Panic disorder should not be given as main diagnosis if pt has depressive disorder

111
Q

PANIC DISORDER

  • bio management options? 2
  • psych management? 1
  • social management? 2

what is 1st line?

A
  • SSRIs (may increase panic initially)
  • propranolol (‘pill in pocket’ - not sure how often used)
  • CBT (go through IAPT)
  • psychoeducation (what to do during a panic attack)
  • reduce alcohol and caffeine intake

1) social
2) psych
3) bio

112
Q

Definition of a phobia? (incl 2 specific types of behaviours that occur)

when do patients present to doctors?

A

–> Anxiety that is situational – initiated by experience or anticipation of phobic stimulus.

Anxiety is restricted to a specific situation or object

Anxiety out of proportion to a specific stimulus

Fear cannot be reasoned or explained away.

ANTICIPATORY behaviour: People tend to worry leading up to the situation: anticipatory anxiety

AVOIDANCE behaviour: Avoidance of stimulus/ situations where it may arise

Patients present when causing loss of function

nb can have panic attacks but not panic disorder as in reaction to a specific stimulus

113
Q

PHOBIAS:

  • what are the 3 main types?
  • norm age of onset and gender split for each?
A

SPECIFIC PHOBIAS

  • F=M
  • childhood

SOCIAL PHOBIA

  • F>M
  • adolescence

AGORAPHOBIA

  • F>M
  • 20-35 years

nb specific phobias are ‘simple’ phobias, whereas social and agoraphobia are ‘complex’ phobias

114
Q

AGORAPHOBIA

  • describe it?
  • what fears does it normally involve? 4
A

Anxiety provoked by open spaces or large crowds where escape is difficult

  • “fear of the marketplace”
  • Fear of space (open or closed)

Usually involve fears of:

  • leaving home / being away from home
  • entering shops, queues
  • crowds and public places, being trapped
  • travelling alone in trains, buses or planes

Avoidance of the phobic situation is often prominent
- Some people with agoraphobia experience little anxiety because they are able to avoid their phobic situations.

Concerns with fainting, dying, not able to escape etc.

115
Q

AGORAPHOBIA

  • what makes it better/more manageable? 2
  • common co-morbid features/diagnoses? 4
A

Better if:

  • way out
  • accompanied e.g. pets

Often occur with panic attacks = panic disorder with agoraphobia

Depressive and obsessional symptoms, and social phobias are also commonly present as subsidiary features

116
Q

SOCIAL PHOBIA

  • describe it?
  • when does it become a problem?
  • what personality features often associated with? 2
  • what can trigger it?
  • what are most common adverse outcomes? 2
A

The fear is of being judged or “scrutinised” by others in a negative way
- can lead to avoidance

This is on a spectrum with normality as everyone has this to a certain extent
- Is only problematic if it interferes with your life and causes you distress

May present as a complaint of blushing, hand tremor, nausea, or urgency of micturition
- Symptoms may progress to panic attacks.

usually associated with low self-esteem and fear of criticism.

May have triggering incident e.g. embarrassing event

ADVERSE OUTCOMES

  • Withdrawal from society
  • Alcohol abuse common to overcome phobia
117
Q

SPECIFIC PHOBIA

  • describe what scared of?
  • risk factors? 2
  • give some examples
A

Specific or simple phobias centre around a particular object, animal, situation or activity

They often develop during childhood or adolescence and may become less severe as you get older.

RISK FACTORS

  • triggering / traumatic event
  • learned phobia from parents/siblings

animal phobias – such as dogs, spiders, snakes or rodents

environmental phobias – such as heights, deep water and germs

situational phobias – such as visiting the dentist or flying

bodily phobias – such as blood, vomit or having injections

sexual phobias – such as performance anxiety or the fear of getting an STI

118
Q

SPECIFIC PHOBIA

  • how majority diagnosed?
  • how most people manage?
  • effect if goes untreated?
A

Phobias are not usually formally diagnosed. Most people with a phobia are fully aware of the problem.

A person will sometimes choose to live with a phobia, taking great care to avoid the object or situation they’re afraid of.

But if you have a phobia, continually trying to avoid what you’re afraid of will make the situation worse.

119
Q

When taking a hx of someone with a simple or complex phobia, what are important things to eillicit? 5

A
  • any triggering/precipitating factors? when did it start?
  • any anticipatory behaviour / anxiety?
  • any avoidance behaviour (or coping, like alcohol)?
  • severity - how severe is it now?
  • psych comorbidities - any GAD, depression, OCD, PTSD etc
120
Q

Important psych differential diagnosis for phobias to screen for? 1

A

PTSD

(also OCD)

(also panic disorder if no trigger, GAD if all the time etc)

121
Q

MANAGEMENT OF PHOBIAS:

  • bio options? 3
  • psych option for specific? 1
  • psych option for agoraphobia + social phobia? 1
  • social management? 2

prognosis?

A

BIOLOGICAL

  • SSRIs (social + agor)
  • propranolol (pill in pocket)
  • benzos (short term - eg if flying)

bio options are just to manage symptoms - real treatment is psych!

PSYCH

  • graded exposure (specific)
  • CBT (complex)

SOCIAL MANAGEMENT

  • psychoeducation: avoidance reinforces the phobia, can use self-help to do own graded exposure
  • reduce alcohol use (esp social phobia)

PROGNOSIS is really good if psychotherapy!

122
Q

DEFINTIONS / DESCRIPTIONS:

  • obsession? (+ common examples)
  • compulsion? (+common examples
A

OBSESSION:
- ideas, images, or impulses that enter the patient’s mind again and again in a stereotyped form
- almost invariably
distressing (because they are violent or obscene, or simply because they are perceived as senseless) and the sufferer often tries, unsuccessfully, to resist them.
- They are, however,
recognized as the individual’s own thoughts, even though they are involuntary and often
repugnant.
- This causes anxiety and distress and leads to an attempt to neutralise ie the compulsion.

Obsessions may include:

  • Fear of getting disease
  • Fear of coming to harm
  • Fear of causing harm to others

nb 90% popn experience obs, difference is that people with OCD can’t dismiss these thoughts as insignificant - they view them as significant and meaningful - eg “Having this thought means that it will come true”

COMPULSION:

  • strong urges to repeatedly perform an action, even though it is unnecessary.
  • stereotyped behaviours that are repeated again and again.
  • not inherently enjoyable, nor do they result in the completion of inherently useful tasks.
  • function is to prevent some objectively unlikely event, often involving harm or danger to or caused by the patient, which they fear might otherwise occur. (ie cancel out the obsessional thought)
  • recognised by the patient as symbolic, pointless or ineffectual
  • Anxiety is almost invariably present.
  • Repeated attempts are made to resist but if acts are resisted the anxiety gets worse

Compulsions can involve:

  • Cleaning/handwashing
  • repeated checking (to ensure that a potentially dangerous situation has not been allowed to develop e.g. turning off electrical switches)
  • orderliness and tidiness.
123
Q

OCD:

  • definition? (incl time frame)
  • four main features / characteristics?
  • common comorbididty/differential?
A

obsessional symptoms or compulsive acts, or both, must be present on MOST DAYS for at least TWO successive WEEKS and be a source of distress or interference with activities.

The obsessive-compulsive symptoms should have the following characteristics:

(a)they must be recognized as the individual’s own thoughts or impulses;

(b)there must be at least one thought or act that is still resisted unsuccessfully, even though others
may be present which the sufferer no longer resists;

(c)the thought of carrying out the act must not in itself be pleasurable (simple relief of tension or
anxiety is not regarded as pleasure in this sense);

(d)the thoughts, images, or impulses must be unpleasantly repetitive

close relationship between
obsessive + compulsive symptoms, particularly obsessional thoughts, and DEPRESSION.

Individuals with
OCD often have depressive symptoms, and patients suffering from recurrent depressive disorder may develop obsessional thoughts during their episodes
of depression. In either situation, increases or decreases in the severity of the depressive symptoms are generally accompanied by parallel changes in the severity of the obsessional symptoms.

Differentiating between OCD and a depressive
disorder may be difficult because these two types of symptoms so frequently occur together.

In an acute episode of disorder, precedence should be given to the symptoms that developed first; when both types are present but neither predominates, it is usually best to regard the depression as primary. In chronic disorders the symptoms that most frequently persist in the absence of the other should be given priority

124
Q

OCD:

  • gender split?
  • main risk factor?
  • norm age of onset?
  • normal course / prognosis?
A

M=F

often prominent
ANANKASTIC (ie OCD) features in the underlying personality

‘anankastic’ is derived from the Greek word meaning ‘compulsion’.

Onset is usually in childhood or early adult
life.

The course is variable and more likely to be chronic in the absence of significant
depressive symptoms.
- compulsions are more easily treatable than the obsessions

nb can subcategorise by if obsessions or compulsions are more prominent or if is mixed

125
Q

Differences between OCD and ankastic or obsessive compulsive personality disorder (OCPD)?

A

1) INSIGHT
- People with OCD are typically distressed by the nature of their behaviors or thoughts, however much they are unable to control them (ie have insight).
- People with OCPD believe that their actions have an aim and purpose. (ie don’t have insight)

2) ADDITIONAL BEHAVIOUR
- OCPD tend to think their way of doing things is the “right and best way,” and they are fixated with following set procedures or routines in their work or daily living, even when these routines are inefficient.
- they tend to be overly controlling of their environments or relationships, wanting others to conform to the strict rules they set.
- maladaptive traits and behaviors, including perfectionism that interferes with completing tasks, rigid following of moral or ethical codes, hoarding behaviors, and an excessive fixation with lists and rules

3) PERVASIVENESS
- OCPD is pervasive in all areas of life (not just with one type of thought or behaviour)

126
Q

OCD:

  • biological options ? 2
  • psychological? 2
A

BIOLOGICAL

  • SSRIs (fluoxetine, sertraline)
  • TCA (clomipramine)

PSYCH

  • CBT sometimes
  • exposure and repeated prevention

eg:

  • stimulus control: patient given specific time of day where allowed to think about obsessional thoughts
  • Apps eg NOCD: pts can watch videos e.g. hand washing, which reduces anxiety and compulsion

Combined psychotherapy AND drug treatment generally better than each alone

127
Q

PTSD:

  • definition?
  • time of onset following event?
  • lifetime prevelence?
A

= Delayed or protracted response to a stressful event or situation (of either brief or long duration) of an exceptionally threatening or catastrophic nature, which is likely to cause pervasive distress in almost anyone

e.g. RTA, severe assault, military violence, rape

Symptoms develop in aftermath –> Approx. 1-6 months after event

(see other flashcard for exactly which type of symptoms needed for diagnosis)

lifetime prevelence = 1-10%

128
Q

PTSD:

  • predisposing risk factors? 3
  • high risk groups of people? 3
  • risk factors of the event/after the event which increase likelihood of developing PTSD? 3
A

PREDISPOSING

  • personality traits (compulsive, asthenic)
  • previous history of neurotic illness
  • genetic: sensitive amygdala & hippocampus – decreased hippocampus size on MRI

nb predisposing factors may lower the threshold for the development of the syndrome or aggravate its course (but they are neither necessary nor sufficient to explain its occurrence)

HIGH RISK GROUPS

  • Traumatic occupation: soldiers, doctors, police
  • disaster victims
  • rape, torture and assault victims = ASK about forensic Hx

RISK FACTORS SURROUNDING EVENT

  • scale of trauma
  • pt’s previous experience
  • low level of social support available
129
Q

PTSD:

  • main groups of symptoms experienced? 5 (incl which needed for diagnosis + which norm found)
  • common psych co-morbidities? 2
A

1) RE-EXPERIENCE (flashbacks, nightmares, intrusive memories)
2) HYPERAROUSAL / anxiety with hypervigilance and increased startle reaction (ie jumpy)
3) NUMBNESS
4) EMOTIONAL BLUNTING / detachment from others
5) AVOIDANCE of activities associated with trauma

(scroll down for comorbidities)

In addition to
evidence of trauma, there must be a repetitive, intrusive recollection or re-enactment of the event in memories, daytime imagery, or dreams.

Conspicuous emotional detachment, numbing of feeling, and avoidance of stimuli that might arouse recollection of the trauma are often present but are not essential for the diagnosis. The autonomic disturbances, mood
disorder, and behavioural abnormalities all contribute to the diagnosis but are not of prime importance.

PSYCH CO-MORBIDITIES

  • depression + suicidal ideation
  • substance misuse
130
Q

Assessment for PTSD:

- things to ellicit from hx?

A
  • Hx of traumatic event
  • when onset of symptoms
  • what symptoms present
  • screen for depression
  • screen for substance misuse
  • RISK ASSESS (esp for suicide!)
131
Q

Management of PTSD:

  • bio option? 1 (be specific)
  • psych option? 1
  • other treatment? 1
  • social? 2

prognosis?

A

BIO
- antidepressants, esp PAROXETINE

PSYCH
- trauma-focused CBT (multiple sessions, repeated graded exposure)

OTHER

  • EMDR (eye movement desensitisation + repreocessing)
  • move eyes side to side whilst thinking of event

SOCIAL

  • treat substance abuse
  • crisis management plan

65% recover in 18 months

132
Q

Adjustment disorder

  • definition? (incl time period)
  • what does it include?
  • common triggers?
A

States of subjective DISTRESS and emotional disturbance, usually interfering with social functioning and
performance, and arising in the period of adaptation to a SIGNIFICANT LIFE CHANGE or to the
consequences of a STRESSFUL LIFE EVENT (including the presence or possibility of serious physical illness).

The stressor may have affected the integrity of an individual’s social network
(through bereavement or separation experiences) or the wider system of social supports and values (migration or refugee status). The stressor may involve only the individual or also his or her group or community.

manifestations vary, and include:
- depressed mood
- anxiety
- worry (or a mixture of these)
- poor concentration
- avoidance / denial
- feeling of inability to cope, plan ahead, or continue in the present situation
- some degree
of disability in the performance of daily routine.
- irritability
- dramatic behaviour or outbursts of violence, but these rarely occur.
- autonommic arousal (eg palpitations and sweating)

However, conduct disorders (e.g.
aggressive or dissocial behaviour) may be an associated feature, particularly in adolescents.

None of the symptoms is of sufficient severity or prominence in its own right to justify a more
specific diagnosis.

In children, regressive phenomena such as return to bed-wetting, babyish
speech, or thumb-sucking are frequently part of the symptom pattern.

The onset is usually within ONE MONTH of the occurrence of the stressful event or life change, and the
duration of symptoms does not usually exceed SIX MONTHS

includes NORMAL GRIEF REACTION

133
Q

Assessment of suspected adjustment disorder:

  • important things to ascertain in hx? 2
  • psychiatric conditions to screen for? 3
A
  • what was the stressful event and when did it happen (ie how long ago)
  • how does it continue to affect life now (incl level of functioning)

SCREEN FOR

  • depression
  • anxiety
  • substance misuse

nb if no stressful event, situation or life change has occured then seek alternative diagnosis - by definition, adjustment disorder requires an event/change to ‘adjust’ to

134
Q

Management of adjustment disorder:

  • treatment aim?
  • psycho? 2
  • social? 2
A

Treatment aims to accelerate the natural process of adjustment.

Working through the life-change reduces anxiety and allows denial and avoidance to decline.

Anxiolytics are rarely required.

PSYCHO

  • problem solving psychotherapy
  • crisis intervention

SOCIAL

  • address any ongoing social issues (eg finances, housing, other relationships etc)
  • address substace misuse

make sure to keep following up

135
Q

abnormal grief reaction/prolonged grief disorder:

  • main features which differentiate between normal and abnormal grief?
  • common symptoms of grief?
A

1) SEVERITY
- increased distress and disability (ie functioning - eg meets criteria for depression)

2) LENGTH
- more than 6 months (can also be delayed in onset)

SYMPTOMS OF GRIEF:

  • longing for deceased
  • emotional pain
  • preoccupation with deceased (incl pseudohallucinations)
  • difficulty accepting death
  • disbelief/denial
  • bitterness/anger
  • blame/guilt
  • feeling one has lost a part of oneself
  • difficulties engaging with acitivities or making plans for the future
  • sadness
  • emotional numbness
  • inability to experience positive mood
  • impairment in important areas of functioning (family, personal, educational, occupational)

nb these symptoms often occur transiently in many bereaved individuals, but it is only when these reactions are experienced on more days than not, causing severe distress and impairment in important areas of functioning more than six months after loss that a PGD diagnosis is applicable

136
Q

abnormal grief reaction:

  • predisposing risk factors?
  • risk factors surrounding death?
A

PREDISPOSING

  • female
  • low educational level
  • insecure attachment + neurotic personalities
  • PMHx of psych conditions

PRECIPITATING

  • very close to deceased (esp partner or child)
  • unnatural / violent death
  • unexpected death
137
Q

abnormal grief reaction:

- important DDx? 2

A
  • depression

- PTSD (esp if they witnessed death)

138
Q

management options for abnormal grief reaction:

  • bio? 1
  • psycho? 2
  • social? 3
A

BIO
- consider antidepressants (but not always)

PSYCHO

  • CBT
  • other psychological therapies

SOCIAL

  • social prescribing etc to increase engagement in social activities
  • address social issues, eg finances, occupation etc
  • specialist grief charities or support groups

nb not a lot of research has been done about this so use clinical judgement

139
Q

describe:

  • 5 stages of grief
  • grief task model
A

One of the most popular models of grief divides it into 5 stages.

1) DENIAL: this may include a feeling of numbness and also pseudohallucinations of the deceased, both auditory and visual. Occasionally people may focus on physical objects that remind them of their loved one or even prepare meals for them
2) ANGER: this is commonly directed against other family members and medical professionals
3) BARGAINING
4) DEPRESSION
5) ACCEPTANCE

It should be noted that many patients will not go through all 5 stages.

The GRIEF TASK MODEL proposes that normal grief is the successful achievement of certain “grief tasks,” whereas complications in managing these tasks might indicate disturbed grief. There is no recommended or specific order in which to achieve these tasks.

Grief tasks include:

  • to accept the reality of the loss
  • to process the associated pain
  • to adjust to a world without the deceased
  • to find an enduring connection with the deceased in the midst of embarking on a new life.
140
Q

Acute stress reaction:

- describe! (incl length)

A

transient disorder
develops in an individual without any other apparent mental disorder in response to exceptional physical and mental stress

Symptoms mixed but could include being in a daze, disorientation, panic, amnesia

Initial state of shock +/- symptoms of anxiety and depression

Symptoms usually appear within MINUTES of the impact of the stressful stimulus /event
- usually subsides within HOURS or DAYS (2-3).

141
Q

MEDICALLY UNEXPLAINED SYMPTOMS, define:

  • somatisation disorder
  • hypochondrial disorder
  • conversion disorder
  • dissociative disorder
  • factitious disorder
  • malingering
A

SOMATISATION DISORDER

  • multiple physical SYMPTOMS present for at least 2 years
  • patient refuses to accept reassurance or negative test results
  • “S for symptoms”

HYPOCONDRIAL DISORDER

  • persistent belief in the presence of an underlying serious DISEASE, e.g. cancer
  • patient again refuses to accept reassurance or negative test results
  • “C for cancer”

CONVERSION DISORDER
- typically involves loss of motor or sensory function
the patient doesn’t consciously feign the symptoms (factitious disorder) or seek material gain (malingering)
- patients may be indifferent to their apparent disorder - la belle indifference - although this has not been backed up by some studies

DISSOCIATIVE DISORDER

  • dissociation is a process of ‘separating off’ certain memories from normal consciousness
  • in contrast to conversion disorder involves psychiatric symptoms e.g. Amnesia, fugue, stupor
  • dissociative identity disorder (DID) is the new term for multiple personality disorder as is the most severe form of dissociative disorder

FACTITIOUS DISORDER

  • aka Munchausen’s syndrome
  • the intentional production of physical or psychological symptoms

MALINGERING
- fraudulent simulation or exaggeration of symptoms with the intention of financial or other gain (eg to get drugs)

142
Q

Somatisation disorders:

- describe the two most common forms of somatisation and the differences between them and how they present?

A

SOMATISATION DISORDERS ARE PSYCHOLOGICAL DISORDERS THAT EXHIBIT PHYSICAL SYMPTOMS OR TAKE THE FORM OF PHYSICAL DISORDERS

There will be no discernable physical or organic reason for these symptoms and this leads to the presumption that they are caused by a psychological disorder

FEATURES OF SOMATISATION
- These physical symptoms do NOT HAPPEN VOLUNTARILY and are completely beyond the individuals control

FORMS OF SOMATISATION
- The two most common forms of somatisation are SOMATISATION DISORDER and HYPOCHONDRIAL DISORDER. The two have distinct psychologies…

In SOMATISATION DISORDER (AKA Briquet’s Syndrome) people have specific symptoms that they complain of
- You can remember this because it’s S for Somatisation and S for symptoms

In HYPOCHONDRIAL DISORDER people are convinced of having a specific disease
- E.g. they think they’ve got cancer or HIV
HYPOCHONDRIACAL DISORDER
- This disorder is distinct form somatisation disorder but it is quite subtly different

In hypochondriacal disorder patients misinterpret normal bodily sensations and think that they mean that they’ve got a serious medical disease.

A key difference from somatisation disorder is:
- Often patients with somatisation disorder will ask for TREATMENT for their symptoms whereas patients with hypochondriacal disorder will become fixated on the diagnosis and so they will often be demanding INVESTIGATIONS to confirm this diagnosis

Again, patients with hypochondriacal disorder will refuse to accept information an reassurance from doctors that there is not organic cause or disease process.

BODY DYSMORPHIC SYNDROME is a specific type of hypochondriacal disorder where people become concerned/preoccupied with a specific defect or deformity e.g. crooked nose or ugly hands.

143
Q

Common symptoms of somatisation disorders:

  • GI?
  • skin?
  • sexual?
  • urinary?
  • neurological?
A

GI:

  • Nausea
  • vomiting
  • diarrhoea
  • food intolerance
  • belching
  • regurgitation
  • abdo pain
  • constipation

Skin:

  • itching
  • burning
  • numbness

Sexual:

  • loss of libido
  • ejaculatory or erectile dysfunction
  • irregular menses
  • dysmenorrhoea

Urinary:

  • dysuria
  • frequency
  • urinary retention

Neurological:

  • paralysis
  • sensory loss
  • hearing or vision loss
  • double vision
  • seizures
  • difficulty swallowing
  • impaired coordination

***patients should have numerous symptoms from more than one of these groups not just one symptom in isolation

144
Q

What complications are people with somatisation disorders at high risk of?

A

Patients with somatisation disorders are highly at risk of having iatrogenic complications - a key example is a patient with multiple abdominal adhesions from having multiple explorative laparoscopies.

145
Q

ICD-10 criteria for somatisation disorder? (incl time frame)

A

In order for somatisation disorder to be diagnosed the ICD-10 suggests all of the following should be present:

  • At least 2 years of symptoms with no organic explanation found
  • Persistent refusal by the patient to accept reassurance from several doctors that there is no physical cause
  • Some degree of functional impairment due to the symptoms and resulting behaviour
146
Q

Describe conversion syndrome

A

This is when there is a perceived loss of motor or sensory function - describes the situation whereby psychological pain is converted into the somatic or physical form.

Often neurological symptoms (paralysis, blindness, sensory loss)

It is distinct from factitious disorder where the patient purposefully feigns symptoms.

In conversion disorder the motor and sensory loss are involuntarily experienced

A common thing that patients seem to experience is profound loss of peripheral visions. They state that is is ‘like looking down a tube’. Again patients might seem indifferent (see below)

Patients might be experiencing La belle Indifference - they might be completely indifferent to their symptoms but this is not well backed up by research

147
Q

Management of somatoform disorders:

  • who to manage?
  • frequency of visits?
  • general principles of management? 6
  • social management to consider? 4
  • psych treatment to consider? 1
  • bio treatment to consider? 1
A

manage by GP

  • see same dr every time to build rapport
  • Schedule frequent visits (e.g. 15 minute visit/month)

This has the effect of:

  • Replaces emergency visits and telephone calls
  • Allows patient attention without a new symptom
  • Not contingent on new complaints
  • Not focused on symptoms

GENERAL PRINCIPLES:

  • Do not suggest “it’s all in your head”
  • Encourage patient to assume active role in their care
  • Focus on function (not symptoms)
  • Manage comorbid psychiatric conditions
  • Provider reassurance that serious conditions have been ruled out (avoid additional lab testing and subspecialist referral)
  • Spend most of the encounter listening (aknowledge that what they are feeling is real, allow the “sick role”)

SOCIAL (focus on stress reduction)

  • exercise
  • manage financial / relationship stress
  • social gatherings / pleasurable activities (incl social prescribing)
  • relaxation (nature walks, yoga)

PSYCH
- CBT (but need to encourage them to come to this idea - eg suggesting pts own strategies haven’t been working - is ‘worth a try’)

BIO
- SSRIs have shown some benefit!

Can also do ‘benign’ management, eg:

  • Hot and cold packs
  • Bandages
  • Canes
  • Lotions
  • Vitamins or Nutritional
  • Supplements
  • Acupuncture
  • Chiropractic massage therapy
  • Biofeedback
148
Q

Definition of a ‘binge’

A

= A ‘binge’ is the consumption of a large number of calories in a relatively short space of time- can be up to 6000 calories in 1-2 hours - with a sense of a ‘loss of control’ while doing this - binge will only end when can find no more food in house or feel in so much pain from all food that have to purge - then get huge guilt and regret and try to purge

Subjective over eating is different!

An episode of binge eating is characterized by
both:
1. Eating in a discrete period of time (e.g. within any 2 hour period), an
amount of food that is definitely larger than what most individuals would
eat in a similar period of time under similar circumstances;
2. A sense of lack of control over eating during the episodes (e.g. a feeling
that one cannot stop eating or control what or how much one is eating).

Binge eating episodes are associated with three or more of the following:

  1. Eating much more rapidly than normal.
  2. Eating until feeling uncomfortably full.
  3. Eating large amounts of food when not feeling physically hungry.
  4. Eating alone because of feeling embarrassed by how much one is eating.
  5. Feeling disgusted with oneself, depressed, or very guilty afterwards.
149
Q

Core psychopathology in eating disorders?

LEARN THIS REALLY WELL

A

1) FEAR of fatness
2) PURSUIT of thinness

3) Body DISSATISAFACTION
4) Body image DISTORTION

5) SELF-EVALUATION based on weight and shape

‘imagine facing your worst fear 3x a day, every day’

150
Q

General psychopathology in eating disorders? 7

ie effect on psychology of being starved

A

when human body is starved, direct effect on psychology:

  • Fixation on food – starvation, restriction
  • Depression
  • Anxiety, social phobia
  • Suicidal ideation
  • OCD symptoms
  • rigidity

Rumination about food, incl:

  • bizarre beliefs about good and bad food, calories, fat etc
  • beliefs around miasmatic calories (thinks can absorb calories through skin or breath them in)
151
Q

Diagnostic criteria for:

  • anorexia nervosa? 3
  • bulemia nervosa? 3
  • binge eating disorder?
A

ANOREXIA NERVOSA

1) BMI <18.5 (or 17.5?)
2) weight loss is self-induced by food-avoidance etc behaviour
3) presence of core psychopathology

BULIMIA NERVOSA

1) BMI >18.5 (or 17.5?)
2) presence of at least one episode of binging and purging per week for at least 3 months
3) presence of core pathology

BINGE EATING DISORDER

1) At least one binge eating episode per week for 3 months
2) marked distress regarding binge eating
3) binge eating is not associated with recurrent use of compensatory/purging behaviours
4) presence of core psychopathology

nb see other flash card for definition of a ‘binge’

ie the presence of CORE PSYCHOPATHOLOGY is present in all eating disorders!!!

152
Q

What are Other Specified Feeding or Eating Disorder (OSFED)

- give examples? 6

A

Symptoms characteristic of a feeing or eating disorder that cause clinical distress or impairment in social, occupational, or other important areas of functioning predominate.

However DO NOT meet the full criteria for any of the disorders in the feeding and eating disorders diagnostic class

ATYPICAL ANOREXIA NERVOSA:
all of the criteria for anorexia nervosa are met, except that despite significant weight loss, the individual’s weight is within or above the normal range.

BULIMIA NERVOSA (of LOW FREQUENCY and/or LIMITED DURATION):
all of the criteria for
bulimia nervosa are met, except that the binge eating and inappropriate
compensatory behaviors occur, on average, less than once a week and/ or for less than 3 months.

BINGE-EATING DISORDER (of LOW FREQUENCY and/or LIMITED DURATION):
all of the criteria for binge-eating disorder are met, except that the binge occurs, on average,
less than once a week and/ or for less than 3 months.

PURGING DISORDER: recurrent purging behavior to influence weight or shape (e.g. self-induced vomiting; misuse of laxatives, diuretics, or other medications) in the absence of binge eating.

NIGHT EATING SYNDROME:
Recurrent episodes of night eating, as manifested by
eating after awakening from sleep or by excessive food consumption after the
evening meal. There is awareness of recall of the eating. The night eating causes significant distress
and/or impairment in functioning.

EXCESSIVE COMPULSIVE EXERCISE
- significant amount of time dedicated to physical activity to detriment of other factors in goal of achieving an unrealistic desired weight

nb don’t need to learn these by heart but just be aware

153
Q

Physical complications of eating disorders:

  • CVS? (2 AN, 2 BN, 1 both)
  • renal? (4 both)
  • GI? (2 AN, 2 BN, 2 both)
  • skeletal? (1 AN, 2 both)
  • endocrine? (3 AN, 1 BN)
  • haem? (2 AN, 1 both)
  • neuro? (2 both)
  • metabolic? (2 both)
  • derm? (2 AN, 1 BN)
A
CVS
AN - Bradycardia
AN - Hypotension
BN - arrythmias
BN - heart failure
BOTH = Sudden death (in AN norm dt QTc prolongation, in BN norm dt arrythmias)
RENAL (all BOTH)
= Oedema (severe in BN)
= Electrolyte abnormalities
= Renal calculi
= Renal failure
GASTROINTESTINAL
AN - Delayed gastric emptying
AN - Nutritional hepatitis
BN - Dental erosion
BN - Oesophageal erosion / perforation
BOTH = Parotid swelling
BOTH = Constipation

SKELETAL
AN - Short stature
BOTH = Osteoporosis
BOTH = Pathological fractures

ENDOCRINE
AN - Amenorrhoea
AN - Infertility
AN - Hypothyroidism
BN - oligo / amenorrhoea

HAEM
AN - Anaemia
AN - Thrombocytopenia
BOTH = Leukopenia (sometimes lymphocytosis in BN)

NEURO (all BOTH)
= Generalised seizures
= Confusional states

METABOLIC (all BOTH)
= Impaired temperature regulation
= Hypoglycaemia

DERM
AN - lanugo
AN - brittle hair and nails
BN - Calluses on dorsum of hands (Russell’s sign)

154
Q

Screening tool used for eating disorders in primary care? 5

when to refer for more detailed assessment?

A

SCOFF Screening:

S = SICK:

do you ever make yourself SICK because you feel uncomfortably full?

C = CONTROL:

do you worry you have lost CONTROL over how much you eat?

O = ONE:

have you lost more than ONE stone in a 3-month period?

F = FAT:

Do you believe yourself to be FAT when others say you are too thin?

F – FOOD:

would you say that FOOD dominates your life?

primary care screening tool (if 2 or more positive, refer for more detailed assessment)

155
Q

What are the two biggest risks with eating disorders? (ie causes of death)

A

1) cardiac arrest / sudden death
2) suicide (20-40% of deaths)

so manage risk of both

anorexia nervosa has highest mortality of any psych condition

156
Q

What are the 4 components for a physical risk assessment in eating disorders?

(ie what 4 things do you need to do/know in order to make an accurate assessment?)

A

1) Clinical Hx and physical examination
2) BMI
3) ECG
4) Bloods (see other flashcards for which ones)

can’t do a valid risk assessment if haven’t got all of these 4 components
- ie don’t base risk on BMI alone!!!

157
Q

Differential diagnoses for anorexia / significant weight loss:

  • psych? 6
  • organic? 4
  • other? 3
A

PSYCH:

  • ANOREXIA NERVOSA
  • FOOD AVOIDANCE EMOTIONAL DISORDER (an ED that affects children which involves a fear of eating which is not accompanied by a fear of weight gain)
  • PHAGOPHOBIA (anxiety disorder characterized by a fear of eating, it is usually initiated by an adverse experience while eating such as choking or vomiting, Individuals with this disorder may present with complaints of pain while swallowing)
    eg Pt saw with liaison with NG tube - had never been good eater then struggled with constipation and was so scared that she’d have it again - scared of the pain
  • DEPRESSION - stop eating and drinking, form of self-neglect
  • ANXIETY - if really anxious - trigger fight or flight, don’t want to eat (similar to post-intensive exercise)
    Somali pt saw with liaison - never eaten much but anxiety just caused her to stop eating- hated how skinny her arms looked (also access to food - dad just having takeaways, didn’t like rice, didn’t know how to eat healthily)
  • PSYCHOSIS / DELUSIONAL BELIEFS - paranoid beliefs of being poisoned

ORGANIC CAUSES:

  • MALIGNANCY - anorexia is a core feature partially due to nausea (also comorbid anxiety or depression)
  • IBD / COELIAC - eat sufficient amounts but not put on weight so may be a clinical suspicion that not eating
  • INFECTIONS - make nauseous - eg flu can shed weight off very healthy people - also TB
  • METABOLIC CONDITIONS - diabetes type 1, hypo/hyperthyroidism, addisons

OTHER

PAST TRAUMA
Pt in becklin who kept throwing up - but because he ate food so quickly, parents had berated him about eating (also parental pressure may -> ED)

AUTISM SPECTRUM DISORDER
Man just ate kidney beans boiled in oil - struggled to put on weight - his autism, restrictive and repetitive behaviour

DRUG ABUSE
Long term cannabis use can suppress appetite (saw on medical show - always take drugs hx - pt didn’t realise this was the reason so didn’t tell staff as thought irrelevant)

158
Q

In addition to a normal psych hx, what additional things to ask about in a person with suspected eating disorder:

  • what groups of behaviours to ask about?
  • what other things to ask about? 5
A

GROUPS OF BEHAVIOURS TO ASK ABOUT:

  • restricting food intake
  • increasing calorie burning
  • suppressing appetite
  • behaviours around obsession over body
  • binge/purge behaviours
  • other social / self harm

(list examples of all of the above on a different flashcard)

OTHER THINGS

  • explore core psychopathology
  • screen for other mental health conditions (eg depression, social anxiety)
  • ask about CVS symptoms (CP, dizziness, palpitations, blackouts)
  • rule out organic DDx (will have other symptoms, and not have core psychopathology)
  • risk assess (physical and DSH/suicide)

also any physical comorbidities - esp insulin-dependent diabetes or pregnancy = very high risk!

159
Q

Common behaviours in eating disorders:

  • restricting food intake? 5
  • increasing calorie burning? 6
  • suppressing appetite? 2
  • obsession over body? 3
  • binge / purge behaviour? 5
  • social behaviours? 4
A

RESTRICTING FOOD INTAKE

  • Dieting
  • Fasting
  • Calorie counting
  • chewing and spitting
  • other culinary behaviour (bizarre combinations of food, spoiling food)

INCREASING CALORIE BURNING

  • Excessive exercise
  • over-activity (fidgeting, twitching, pacing, fetching one thing at a time)
  • Thyroxine
  • Diet pills
  • ritalin
  • cooling (wearing less clothes)

SUPPRESSING APPETITE

  • Water loading
  • Appetite suppressants (gum, cigarettes)

OBSESSION OVER BODY

  • Excessive weighing
  • body checking
  • mirror gazing or avoidance

BINGE / PURGE BEHAVIOUR

  • binging
  • purging
  • starve-bing-purge cycle
  • misuse of insulin
  • laxatives

SOCIAL

  • Avoidance
  • Isolation
  • deliberate self-harm (DSH)
  • Substance misuse.
160
Q

Physical examination for eating disorder:

  • obs that may be abnormal? 3
  • other physical examination things to look for / test? 6
A

OBS

  • bradycardia
  • hypotension
  • hypothermia

OTHER

  • irregular pulse
  • postural hypotension
  • enlarged salivary glands
  • dentition
  • skin on knuckles
  • test for proximal myopathy

basically do a full examination but look out for these specific things

161
Q

How do you test a patietn with suspected ED for proximal myopathy?

A

SQUAT TEST

get them to squat right down and then stand up (without using arms)

if can’t do, concerned about protein malnutrition

162
Q

BMI and risk:

  • cut off for anorexia?
  • cut off for moderate risk?
  • cut off for high risk?

what to be aware of when measuring BMI?

A

BMI = wt(kg)/ht(m)2

<18.5 = AN (?changed to 17.5)

<15 = moderate risk

<13 = high risk (start seeing organ failure)

Proxy measure of physical risk with clear limitations
- can be manipulated, eg water loading, so don’t use in isolation

163
Q

Possible ECG findings in eating disorders? 3

how to manage these abnormalities?

A
  • Bradycardia (<40bpm)
  • t wave changes (hypokalaemia)
  • QTc prolongation (>450ms)

only way to reverse these abnormalities is adequate nutrition, don’t need cardiac drugs

most deaths due to cardiac arrest.
- nb Cardiac abnormalities in up to 85% of anorexia pts.

164
Q

Eating disorder:

  • initial bloods to get? 8
  • findings?

(way to remember findings)

A

standard

  • FBC
  • U+E
  • LFT
  • glucose

more specialist

  • CK
  • phos, Mg, Ca
  • TFTs
  • Zn

(^get all of these, just split them up so easier to remember)

also in anorexia:
- low FSH, LH, oestrogens, testosterone
- raised cortisol + growth hormones
- imppaired glucose tolerance
- hypercholestoraemia
- hypercarotenaemia (raised beta-carotene)
low T3 (if not taking thyroxine)

“in ANOREXIA = most things low BUT G’s and C’s raised: growth hormone, glucose, salivary glands, cortisol, cholesterol, CK, carotinaemia”

165
Q

blood findings in eating disorder behaviours:

  • starvation? 1
  • vomiting? 1
  • water-loading? 1
  • laxatives misuse? 2
  • diuretics misuse? 2
  • thyroxine misuse? 2

other findings in bloods:

  • bone marrow hypoplasia? 2
  • proximal myopathy? 2
A

bloods can show what behaviours patients have been doing - even if they don’t admit it

Starvation
= Hypoglycaemia

Vomiting
= Hypokalaemia

Water-loading
= Hyponatraemia

Laxative misuse
= Hyperkalaemia + Hyponatraemia

Diuretics misuse
= Hypokalaemia, hyponatraemia

Thyroxine misuse
= ↑T3/T4, ↓TSH

Bone marrow hypoplasia
= Normocytic anaemia + Leucopenia

Proximal myopathy
= ↑ CK, ↑ LFTs

166
Q

blood findings in refeeding syndrome?

A

Re-feeding syndrome

  • Hypophosphataemia
  • Hypomagnesaemia
  • Hypocalcaemia
  • Hypokalaemia

so low of all the positive ions (except sodium)

167
Q

Management of eating disorders:

  • where most managed?
  • referral criteria (in yorkshire) for specialist ED unit? 3
  • other hospital wards sometimes found/managed in?
A

Usually managed as outpatient

  • Most AN require specialist Rx
  • BN + OSFED/EDNOS will mainly be treated either in primary care or secondary services

SPECIALIST ED UNIT (yorkshire)

  • moderate/severe AN (bmi <17)
  • severe BN (daily bingeing + purging)
  • OSFED/EDNOS with comorbid IDDM or pregnancy

OTHER WARDS:

  • Gastro-enterology ward (for initial refeeding)
  • Acute medical ward (if electrolyte disturbances)
  • General psychiatric ward (avoid if possible!!!)

Match needs to appropriate setting

  • clinical state, level of risk
  • local resources
  • capacity to consent to treatment
  • motivation to change
  • patient and family’s views
168
Q

Name of MDT group who normally manage people with severe AN?

- who is in this group?

A

MARSIPAN = Management of really sick patients with anorexia nervosa

MARSIPAN group:

  • Physician with special interest in ED
  • ED/Liaison Psychiatrist +/- CMHT
  • Dietician and nutrition support team
  • Nursing team

Have regular MARSIPAN meetings
- coordinated care

169
Q

MAIN STAGES OF MANAGEMENT OF EATING DISORDERS? 3

in which order should they be done?

(don’t describe in detail - on other flashcard)

A

1) nutrition (if super low)
2) motivational enhancement
3) formal psychological support

170
Q

1st stage of ED management: NUTRITION:

  • why do this first?
  • who mainly involved?
  • what teach? (1 to all, 1 to BN)
  • last resort?
A

because when low weight, psychology is more rigid, need to increase nutrition first before can manage psychopathology

use dieticians a lot

EDUCATION on what body needs, eg:

  • carbs at every meal to stabilise blood sugar
  • dairy 2-3x a day for clcium, vit D and phosphate
  • protein 2-3x a day for growth + repair (incl immune systm, skin integrity, tissue restoration)

explaining the binge-purge cycle
- explain to BN that need regular meals to avoid purging (typically binges happen later in the day)

NG feed as last resort

171
Q

2nd stage of management of eating disorders: MOTIVATIONAL ENHANCEMENT

  • what is this? what’s the aim? why necessary?
  • what sort of questions to ask?
  • how long?
A
  • before any in-depth psychotherapy
  • most pts are initially not distressed - it’s their relative who are
  • At point of referral they don’t realise that they have a problem

AIM

  • to understand the psychological functions of the ED
  • weigh up how the individual feels about change
  • to increase motivation to change (lots of motivational interviewing techniques)
  • to make an informed decision about whether this is the right time to change
TECHNIQUES / Qs:
= Affirmations
= Reflective listening
= Open ended questions
--- Why Change?
--- Advantages/Disadvantages?
--- What do you like about your ED?
--- What do you dislike about your ED?
--- Write a list of all the things you have stopped doing because of your ED
--- Barriers to change – What would stop me giving up your ED?
= Summaries
= Elicit change talk

Brief therapy, usually lasting 4-6 sessions
- Beneficial in the initial phase of treatment with promotion of responsibility and a commitment to change

172
Q

What are the steps of the change cycle? 7

what % of people referred to specialist ED are at action phase?

A

1) PRE-CONTEMPLATION
- unwilling to change

2) CONTEMPLATION
- considering change

3) DETERMINATION
- preparing to change

4) ACTION
- implementing change

5) MAINTENANCE
- maintaining change

6) either PERMANENT EXIT or RELAPSE

At point of referral they don’t realise that they have a problem (<50% referred to specialist ED are in action phase)

173
Q

3rd stage of eating disorder management: FORMAL PSYCH SUPPORT

  • best options for AN? 3
  • best options for BN? 2
  • main initial management of OSFED?
  • what medication management is sometimes considered? and for what?
A

ANOREXIA NERVOSA

  • MANTRA (maudsley approach to treating anorexia)
  • CBT
  • SSCM (specialist supportive counselling)

nb all of equal efficacy, try others if don’t work

BULIMIA NERVOSA

  • CBT
  • IPT

nb both of equal efficacy, try others if don’t work

nb BN much easier to treat than AN

ALSO can use FLUOXETINE 60mg daily in BN (not in AN, nb dose is higher than for depression)

OSFED = mainly GUIDED SELF-HELP

  • Education (e.g. physical and psychological risks of disordered eating)
  • Increased awareness and responsibility (e.g. food/thought diaries)
  • Structured eating
  • Identify and challenge beliefs
  • Promote self-management skills
174
Q

What can primary care doctors do to help in the management of eating disorders? 8

A
  • Develop therapeutic alliance
  • Diagnosis & screening
  • Risk assessment and coordinate medical monitoring (BMI, bloods, ECG)
  • Medication review
  • Use motivational enhancement techniques
  • Knowledge of local care pathways and MARSIPAN
  • Guided self help and support groups
  • Encourage self-monitoring (e.g. diaries) and structured eating

*see my full psych notes for additional resources for GPs